LSAT and Law School Admissions Forum

Get expert LSAT preparation and law school admissions advice from PowerScore Test Preparation.

 Administrator
PowerScore Staff
  • PowerScore Staff
  • Posts: 8929
  • Joined: Feb 02, 2011
|
#59051
Please post your questions below!
 Moris_cn
  • Posts: 5
  • Joined: Oct 21, 2018
|
#59753
Can someone explain to me how we should analyze this question? Thanks!
 Adam Tyson
PowerScore Staff
  • PowerScore Staff
  • Posts: 5191
  • Joined: Apr 14, 2011
|
#59804
As with most LR questions, Moris_cn, the best place to start is by identifying the conclusion. What is the author trying to prove here? In this case, it's the first sentence - wages will be protected by such a law.

Next, identify what evidence is being used to try to support that conclusion. The author says that a law capping executive pay at 50x the pay of the lowest paid employees will motivate those execs to not cut employee pay.

Now, we need to see what the gap is between that evidence and that conclusion. The assumption will fill that gap. How does the author get from that evidence to that conclusion? What must he believe in order to make that leap?

Here, I see two assumptions, two big gaps. First, the author has to assume that there are at least some companies were the execs are already making at least 50x what their lowest paid employees make! What if there aren't? What if all execs currently make only 30x what the lowest paid employee does? Then this cap won't be a big deal, and they could still cut that employee pay and stay within the law. Second, the author assumes that the execs will actually obey the law! What if they won't? Then wages could still be cut.

In sum, start by identifying the conclusion, then by identifying the evidence, and finally looking for the gap between them. It can be a very mechanical process. Give that a try and let us know how it works!
 tizwvu34
  • Posts: 20
  • Joined: Aug 15, 2016
|
#65021
Adam Tyson wrote:Here, I see two assumptions, two big gaps. First, the author has to assume that there are at least some companies were the execs are already making at least 50x what their lowest paid employees make! What if there aren't? What if all execs currently make only 30x what the lowest paid employee does? Then this cap won't be a big deal, and they could still cut that employee pay and stay within the law.


Adam,
Is this actually a necessary assumption of the argument because the stated number is a purely hypothetical number thrown out there as indicated by the word "say"? It seems to me that the economist is saying, "the wages would be protected by a maximum wage law," but then kind of shoots from the hip as indicated by "say, 50 times." Wouldn't we need it stated hard and fast that a 50x limit would protect in order to allow us to assume that there are already execs making at least 50x what their lowest employees make?

Thanks for the clarification.
 LSAThangman
  • Posts: 7
  • Joined: Dec 26, 2018
|
#65169
One thing I don't understand is how negating E) precludes the conclusion -- from how I read it, the conclusion should actually survive if we negate E). If we say that at least one corporate cuts the pay of lowest-paid employees, the conclusion can still survive because it stipulates "the wages of many" such workers would be protected, not "all" of them.

Further, I don't understand why D) isn't correct. The author seems to imply that one major reason corporate executives cut pay is to increase their own pay/benefits by increasing corporate profit. If we negate D), we could have a scenario where executives cut pay to increase corporate benefits, REGARDLESS of whether their own pay increases.

Please let me know if I'm missing something here.
 Adam Tyson
PowerScore Staff
  • PowerScore Staff
  • Posts: 5191
  • Joined: Apr 14, 2011
|
#65179
tizwvu34, You are correct that the specific number here is just an example and is not a fixed aspect of the argument. If we ignore the number we still end up with the same analysis - the author thinks an earnings cap on executives would protect the wages of many employees. He has to be assuming that if an executive earnings cap of some kind was in place, at least some executives would refrain from cutting employee wages. It doesn't matter whether that cap is 50x or 30x or 10000x - he still has to assume that at least one executive would be motivated by the cap to refrain from cutting those wages.

LSAThangman, I suspect from your question that you may have incorrectly negated answer E to this question. To negate a claim that one or more would not cut wages, we don't say that one or more WOULD cut wages. The reason this is not the proper negation is that both statements are consistent - they can both be true. For a correct negation, you need an answer that is the logical opposite of the answer, which makes the original answer false. To negate "one or more would not" we instead say "none would not", which, due to the double negative, is the same as saying "all would." The negation of answer E is "If such a maximum wage law were enacted in the economist’s country, ALL corporate executives WOULD cut the pay and benefits of their corporations’ lowest-paid employees in the economist’s country." That destroys the argument that such a law would protect those wages, and thus proves that E is a necessary assumption of the argument. The author must assume that at least some executives would choose not to cut worker wages under the new law.

Does the author have to assume that the executives would never change the wages of the lowest paid employees, as stated in answer D? Not at all - he would be perfectly fine with those wages being increased. "No change" is too broad an idea here, and the author does not have to assume no change. He only has to assume that some employees will not have their wages reduced.
 LSAThangman
  • Posts: 7
  • Joined: Dec 26, 2018
|
#65215
Dear Mr. Tyson,

I did not realize that I should have negated E) as you did. When you put it like that, it makes perfect sense. Further, I thank you for pointing out the distinction between "change" and "cut" for D) -- I missed that difference when I took the test. This question makes sense now.

Thank you again!
 180bound
  • Posts: 34
  • Joined: Jun 11, 2019
|
#75254
I’m having trouble understand the wording in this question. The conclusion states that the law would remove the incentive. But to me removing the incentive to something doesn’t at all guarantee an outcome. I’m a teacher to second graders and I remove incentives for behavior all the time but never does that mean the kids I work with will automatically behavior differently because of such. The same goes with adults to me.

Thanks!
 _klkl
  • Posts: 4
  • Joined: May 11, 2020
|
#75391
180bound wrote:I’m having trouble understand the wording in this question. The conclusion states that the law would remove the incentive. But to me removing the incentive to something doesn’t at all guarantee an outcome. I’m a teacher to second graders and I remove incentives for behavior all the time but never does that mean the kids I work with will automatically behavior differently because of such. The same goes with adults to me.

Thanks!
I also had the same thought, and realized after that this is exactly the assumption required by the stimulus.

The economist says that the law would remove one incentive, which will in turn protect employees. Basically the economist is assuming that if this specific incentive is removed, some CEOs would actually change their behavior.

So that brings us to E), which says:
If the law is enacted (i.e. this incentive erased ) :arrow: at least some CEOs will change their ways

Basically without this assumption, the conclusion doesn't hold and nothing would change, as you say. Seems idealistic and clearly the skeptic was repelled by the right answer choice, but it's one economist's opinion.
 Christen Hammock
PowerScore Staff
  • PowerScore Staff
  • Posts: 61
  • Joined: May 14, 2020
|
#75461
This is exactly right! :) One of the assumptions of the economist's argument is that removing the incentive will result in at least some corporate executives changing their behavior. If CEOS didn't respond to the change, then the law would not have the effect of protecting "many" of the lowest-paid employees. Answer Choice (E) captures that assumption.

Get the most out of your LSAT Prep Plus subscription.

Analyze and track your performance with our Testing and Analytics Package.